Proving Countable Infinite Accumulation Points in a Set

Click For Summary
A set with a countable infinite number of accumulation points can be represented by s = {k + 1/n | k ∈ integers, n ∈ natural numbers}. The definition of an accumulation point states that every neighborhood of a point contains infinitely many elements of the set. The discussion highlights the confusion regarding the nature of rational numbers and their density in the real numbers, clarifying that while the rationals are dense, not all points in R are accumulation points. The example provided, {k + 1/n}, is confirmed to be a valid set with the desired properties. This illustrates the concept of countable infinite accumulation points effectively.
IntroAnalysis
Messages
58
Reaction score
0

Homework Statement



Give an example and prove there is a set with a countable infinite set of accumulations points.

Homework Equations


An example would be s = {k + 1/n l k element integers, n element natural numbers}

integers are countable infinitie a bijection exists with natural numbers

Def: Let S be a set of real numbers. A, element reals, is an accumulation point iff every neighborhood of A contains infinitely many elements of S.

Def: Let x element reals. Then a set Q, subset Reals, is called a neighborhood of x iff there exists epsilon > 0 such that (x -e, x + e) is a subset of Q.


The Attempt at a Solution



I've spent hours and don't know how to start to prove this. Would appreciate any help!
 
Physics news on Phys.org


Do you know a set that's countable and dense in R? What about that set?
 


Rationals are countable and dense in R. Still not sure where I am to take this.
 


Aren't the rationals a set consisting of all accumulation points?
 


micromass said:
Aren't the rationals a set consisting of all accumulation points?

Are you thinking about rationals in Q? If you are thinking about rationals in R, then all points of R are accumulation points. That's not countable. What's wrong with {k+1/n}?
 


Oh my, it appears I've been reading the questio entirely wrong :blushing:

Yep {k+1/n | k,n naturals} are fine!
 
Question: A clock's minute hand has length 4 and its hour hand has length 3. What is the distance between the tips at the moment when it is increasing most rapidly?(Putnam Exam Question) Answer: Making assumption that both the hands moves at constant angular velocities, the answer is ## \sqrt{7} .## But don't you think this assumption is somewhat doubtful and wrong?

Similar threads

Replies
1
Views
2K
  • · Replies 25 ·
Replies
25
Views
3K
  • · Replies 11 ·
Replies
11
Views
3K
  • · Replies 5 ·
Replies
5
Views
2K
  • · Replies 5 ·
Replies
5
Views
2K
  • · Replies 3 ·
Replies
3
Views
902
  • · Replies 3 ·
Replies
3
Views
1K
Replies
3
Views
2K
Replies
5
Views
3K
  • · Replies 18 ·
Replies
18
Views
3K